#1
|
||||
|
||||
TMO 16 Discussion
ข้อสอบวันแรก
1. ให้ $ABCDE$ เป็นรูปห้าเหลี่ยมนูน ซึ่ง $\angle AEB = \angle BDC = 90^\circ $ และ $\overline{AC} $ แบ่งครึ่งมุมทั้ง $\angle BAE$ และ $\angle DCB$ ให้วงกลมล้อมรอบรูปสามเหลี่ยม $ABE$ ตัดเส้นตรง $AC$ อีกครั้งที่จุด $P$ (ก) จงแสดงว่า $P$ เป็นจุดศูนย์กลางของวงกลมล้อมรอบรูปสามเหลี่ยม $BDE$ (ข) จงแสดงว่า จุด $A,C,D,E$ อยู่บนวงกลมเดียวกัน หมายเหตุ รูปห้าเหลี่ยมนูน คือ รูปห้าเหลี่ยมที่มุมภายในแต่ละมุมมีขนาดน้อยกว่า $180^\circ$ 2. ให้ $a$ และ $b$ เป็นจำนวนเต็มบวกที่แตกต่างกัน ถ้า $a$ และ $b$ เป็นจำนวนเฉพาะสัมพัทธ์ แล้ว จงแสดงว่า $\frac{2a(a^2+b^2)}{a^2-b^2}$ ไม่เป็นจำนวนเต็ม 3. จงหาฟังก์ชัน $f: \mathbb{R^+} \rightarrow \mathbb{R^+}$ ทั้งหมดซึ่ง $f(x+yf(x)+y^2)=f(x)+2y$ สำหรับทุก $x,y \in \mathbb{R^+}$ 4. กระต่ายตัวหนึ่งเริ่มต้นอยู่ที่ตำแหน่ง $0$ และกระโดดไปมาบนเส้นจำนวนจริง ในการกระโดดแต่ละครั้ง กระต่ายจะกระโดดไปยังตำแหน่งที่เป็นจำนวนเต็มตำแหน่งใดก็ได้ แต่ต้องไม่กระโดดอยู่กับที่ ให้ $N(a)$ แทนจำนวนวิธีที่กระต่ายจะกระโดดให้ได้ระยะทางรวมทั้งหมด $2019$ หน่วย แล้วสิ้นสุดที่ตำแหน่ง $a$ จงหาจำนวนเต็ม $a$ ทั้งหมด ซึ่ง $N(a)$ เป็นจำนวนคี่ 5. กำหนดให้ $a,b,c$ เป็นจำนวนจริงบวกซึ่ง $abc=1$ จงแสดงว่า $$\frac{4a-1}{(2b+1)^2}+\frac{4b-1}{(2c+1)^2}+\frac{4c-1}{(2a+1)^2} \ge 1$$ ข้อสอบวันที่ 2 6. จงหาฟังก์ชัน $f:\mathbb{R}\rightarrow \mathbb{R}$ ทั้งหมดซึ่ง $xf(y)+yf(x) \le xy$ สำหรับทุก $x,y \in \mathbb{R}$ 7. ให้ $A=\{-2562,-2561,...,2561,2562\}$ จงแสดงว่า สำหรับฟังก์ชันหนึ่งต่อหนึ่งทั่วถึง $f:A \rightarrow A$ $\sum_{k=1}^{2562}\left|\, f(k)-f(-k)\right|$ มีค่ามากสุด ก็ต่อเมื่อ $f(k)f(-k)<0$ สำหรับทุก $k=1,2,...,2562$ 8. ให้ $ABC$ เป็นรูปสามเหลี่ยม โดยที่ $AB \not= AC$ และมี $\omega$ เป็นวงกลมล้อมรอบ ให้ $I$ เป็นจุดศูนย์กลางของวงกลมแนบในรูปสามเหลี่ยม $ABC$ ซึ่งสัมผัสด้าน $BC$ ที่จุด $D$ ให้วงกลมซึ่งมี $\overline{AI}$ เป็นเส้นผ่านศูนย์กลาง ตัดวงกลม $\omega$ อีกครั้งที่จุด $K$ ถ้าเส้นตรง $AI$ ตัดวงกลม $\omega$ อีกครั้งที่จุด $M$ จงแสดงว่า $K,D,M$ อยู่บนเส้นตรงเดียวกัน 9. รูปไชยศรี คือ รูปสามเหลี่ยมที่มีจุดยอดทั้งสามเป็นจุดยอดของรูป 2019 เหลี่ยมด้านเท่ามุมเท่ารูปใดรูปหนึ่ง โดย รูปไชยศรี ที่ต่างกัน อาจเกิดจากรูป 2019 เหลี่ยมด้านเท่ามุมเท่าขนาดต่างกัน รูปทับแก้ว คือ รูปหลายเหลี่ยมนูนซึ่งสามารถตัดแบ่งทั้งรูป ออกเป็น รูปไชยศรี หลาย ๆ รูป โดยที่จุดยอดของ รูปไชยศรี ไม่จำเป็นต้องอยู่บนเส้นขอบของรูปหลายเหลี่ยมนูนนี้ จงหาว่า รูปทับแก้ว มีจำนวนเหลี่ยมได้มากสุดเท่าใด หมายเหตุ รูปหลายเหลี่ยมนูน คือ รูปหลายเหลี่ยมที่มุมภายในแต่ละมุมมีขนาดน้อยกว่า $180^\circ$ 10. จงแสดงว่ามีจำนวนคี่บวก $n$ อยู่อนันต์จำนวนที่ทำให้ $n!+1$ ไม่เป็นจำนวนเฉพาะ 02 มิถุนายน 2019 11:55 : ข้อความนี้ถูกแก้ไขแล้ว 1 ครั้ง, ครั้งล่าสุดโดยคุณ NaPrai |
#2
|
|||
|
|||
มาลงข้อ 7 แบบที่ยากขึ้นนะครับ
ให้ $A = \{-2562,-2561,...,2561,2562\}$ หาจำนวนฟังก์ชั่น $f : A^{2562} \rightarrow A^{2562}$ ซึ่ง $f$ 1 ต่อ 1 และทั่วถึง ที่ $$\sum_{i_1=1}^{2562}\sum_{i_2=1}^{2562}...\sum_{i_{2562}=1}^{2562} |f(i_1,i_2,...,i_{2562})-f(-i_1,-i_2,...,-i_{2562})|$$ มีค่าสูงสุด เมื่อ $|(a_1,a_2,...,a_{2562})-(b_1,b_2,...,b_{2562})| = |a_1-b_1| + |a_2-b_2|+...+|a_{2562}-b_{2562}|$ 20 พฤษภาคม 2019 23:08 : ข้อความนี้ถูกแก้ไขแล้ว 5 ครั้ง, ครั้งล่าสุดโดยคุณ Lspeed |
#3
|
||||
|
||||
อ้างอิง:
|
#4
|
|||
|
|||
ใช่แล้วครับคุณ NaPrai แต่ Key Idea หลักก็แทบจะเหมือนเดิมเลยครับ ซึ่งถูกสปอยไปในข้อ 7 หมดแล้วครับ ถถถ
|
#5
|
|||
|
|||
ผมว่าข้อ 5 ปีนี้ค่อนข้างง่ายนะครับ รู้สึกว่าข้อ 4 ยากกว่าข้อ 5 อีก
|
#6
|
||||
|
||||
อ้างอิง:
//ส่วนตัวคิดว่า 4 กับ 5 เป็นข้อที่พอ ๆ กัน 21 พฤษภาคม 2019 00:08 : ข้อความนี้ถูกแก้ไขแล้ว 2 ครั้ง, ครั้งล่าสุดโดยคุณ NaPrai |
#7
|
||||
|
||||
ข้อ 6 ทำยังไงกันครับ
แสดงได้ไม่ยากว่า $f(0)=0$ ต่อไปนี้จะพิจารณาเฉพาะกรณี $x, y$ ไม่เป็น $0$ ให้ $f(x)=xg(x)$ ได้ $xyg(y)+xyg(x)\leqslant xy$ แทน $y$ ด้วย $x$ ได้ $g(x)\leqslant \dfrac{1}{2} $ ทุก $x\in \mathbb{R} $ แทน $y$ ด้วย $-x$ ได้ $g(x)+g(-x)\geqslant 1$ แต่ $g(x)\leqslant \dfrac{1}{2} $ ทุก $x\in \mathbb{R} $ ดังนั้น $g(x)=\dfrac{1}{2} $ จะได้ $f(x)=\dfrac{x}{2} $ อยากรู้ว่าถ้าไม่เปลี่ยน f เป็นฟังก์ชันอื่น (อย่างที่ผมทำคือเปลี่ยนเป็น g) จะทำข้อนี้ได้มั้ย
__________________
เหนือฟ้ายังมีอวกาศ |
#8
|
|||
|
|||
Solution ทั้ง 10 ข้อ (ภาษาอังกฤษนะครับ พอดีพิมพ์เก็บไว้แค่อังกฤษ)
Clearly $P$ is the projection from $B$ to $AC$ thus quadrilaterals $ABPE$ and $CBPD$ are cyclic. This implies that $PB=PD=PE$, completing (i). For (ii), we just observe that $$\angle CAE + \angle CDE = \angle PBE + (90^{\circ} + \angle BDE) = 180^{\circ}.$$ The main claim is Claim: (Gcd bash) We have $\gcd(a^2-b^2, a(a^2+b^2))\mid 2$. Proof: Let $d\mid a^2-b^2$ and $d\mid a(a^2+b^2)$. We get \begin{align*} d\mid a(a^2+b^2) + a(a^2-b^2) &\implies d\mid 2ab^2 \\ d\mid 2a(a^2+b^2) &\implies d\mid 2a^3 \\ d\mid a^2-b^2\mid 2(a^4-b^4) &\implies d\mid 2b^4 \end{align*} thus $d\mid 2\gcd(a,b)^4=2$ so we are done. By the claim, we get $a^2-b^2\mid 4$ which is a clear contradiction. The answer is $f(x)=2\sqrt{x+c}$ for any constant $c\geqslant 0$. It's straightforward to verify that they are all work. Now we present several solutions which show that these are all possible solutions. Solution 1 (Substitution, Pitchayut): Plug in $(x,y) = \left(a, \frac{\sqrt{f(a)^2+4b}-f(a)}{2}\right)$ gives $$f(a+b) = \sqrt{f(a)^2+4b} \implies f(a+b)^2=f(a)^2+4b$$ This is enough to conclude that $f(x)^2 = 4x+c$ for some constant $c$. This implies the set of solutions mentioned above. Solution 2 (Squaring, Nithid): Squaring the entire equation gives $$f(x+yf(x)+y^2)^2 = f(x)^2 + 4(yf(x)+y^2).$$ By Intermediate Value Theorem, function $g(y) := yf(x)+y^2$ is surjective on $\mathbb{R}^+$ as $\lim\limits_{y\to 0} g(y)=0$ and $\lim\limits_{y\to\infty} g(y)=\infty$. Thus we get $$f(x+t)^2 = f(x)^2+4t$$ for any $x,t\in\mathbb{R}^+$. This is enough to conclude the solution. Solution 3 (Injectivity): First, we prove that $f$ is injective. Suppose that $f(x+t)=f(x)$ for some $x,t\in\mathbb{R}^+$. Arguing as in above solutions, we can choose appropriate $y$ such that $yf(x)+y^2=t$. Using this choice of $x,y$ in the equation gives $y=0\implies t=0$ which is contradiction. Now it's easy to finish. Plug in $y=\tfrac{f(t)}{2}$ gives $$f\left(x+\frac{f(x)f(t)}{2} + \frac{f(t)^2}{4}\right) = f(x)+f(t) = f\left(t+\frac{f(x)f(t)}{2} + \frac{f(x)^2}{4}\right).$$ By injectivity, $4x+f(t)^2 = 4t+f(x)^2$ so we are done. Solution 1 (Generating Function, Pitchayut): Consider the quantity $$T = (x+x^2+x^3+...)+(y+y^2+y^3+...) = \frac{x}{1-x}+\frac{y}{1-y}$$ and define generating functions $$F(x,y) = 1+T+T^2+...$$ It's clear that the coefficient of $x^ay^b$ in $F$ equals to the number of ways to jump with a total distance of $a+b$ and arrive at position $a-b$. (i.e. variable $x$ corresponds to positive jumps and variable $y$ corresponds to negative jumps). Now we evaluate $F(x,y)$ in $\pmod 2$. To do this, let $G(x,y) = 1-T+T^2-T^3+...$ so that $G\equiv F\pmod 2$ and $$G(x,y) = \frac{1}{1+T} = \frac{1}{1+\frac{x}{1-x}+\frac{y}{1-y}} = \frac{(1-x)(1-y)}{1-xy}$$ Thus, we have $$G(x,y) = (1-x-y+xy)(1+(xy)+(xy)^2+(xy)^3+...)$$ It's clear that all odd coefficients are in form $x^ny^{n+1}$ and $x^{n+1}y^n$, which corresponds to $N(1)$ and $N(-1)$. Thus the answer is $\boxed{\{1,-1\}}$. Solution 2 (Combinatorial, Official): Encode each positive jump by the corresponding number of $\texttt{+}$ and encode each negative jump by the corresponding number of $\texttt{-}$. We also seperate each jump by $\texttt{|}$. For instance, $$\texttt{++|+++|---}\implies 0\stackrel{+2}{\longrightarrow} 2\stackrel{+3}{\longrightarrow} 5\stackrel{-3}{\longrightarrow} 2.$$ Clearly, in $N(a)$, we must have $\frac{2019+a}{2}$ $\texttt{+}$'s and $\frac{2019-a}{2}$ $\texttt{-}$'s. We also note that a $\texttt{|}$ must be inserted between $\texttt{+-}$. Now, fix a sequence consisting many $\texttt{+}$ and $\texttt{-}$. Call a sequence \emph{bad} if and only if there are odd number of ways to insert $\texttt{|}$. Claim: The only bad sequences are $\texttt{+-+-+...+}$ and $\texttt{-+-+-...-}$. Proof: If $m,n$ denote the number of consecutive $\texttt{++}$ and $\texttt{--}$ respectively. Then clearly the number of ways to insert $\texttt{|}$ is precisely $2^{m+n}$. Thus the sequence is bad if and only if there are no $\texttt{++}$ and $\texttt{--}$ at all so we are done. The two bad sequences correspond to $N(1)$ and $N(-1)$ thus the answer is $\{1,-1\}$. Add one to each term and divide by $4$. This is equivalent to $$\sum_{\mathrm{cyc}}\frac{b^2+b+a}{(2b+1)^2}\geqslant 1$$ Now we can use Cauchy Schwarz in form $(b^2+b+a)(1+b+\frac{1}{a})\geqslant (b+b+1)^2$. Thus it suffices to prove that $$\sum_{\mathrm{cyc}}\frac{1}{b+\frac{1}{a}+1}\geqslant 1.$$ In fact, it turns out to be an equality. The cleanest way to verify that is to substitute $a=\frac{x}{y}, b=\frac{z}{x}, c=\frac{y}{z}$ and see that each term is equal to $\frac{x}{x+y+z}$. The answer is only $f(x)=\tfrac{x}{2}$, which clearly works. Define $g(x) = f(x)-\frac{x}{2}$. Then the given equation is equivalent to $xg(y)+yg(x)\leqslant 0$. We aim to show that $g\equiv 0$. Plugging in $x=y$ gives $xg(x)\leqslant 0$. Thus plugging in $y=-x$ gives $$xg(-x) + \underbrace{(-xg(x))}_{\geqslant 0} \leqslant 0\implies xg(-x)\leqslant 0$$ Replacing $x$ by $-x$ gives $xg(x)\geqslant 0$ for any $x\in\mathbb{R}$. This is enough to conclude that $g(x)=0$ for any $x\ne 0$. Seeking $g(0)$, we just drop $x=0$ and we are done. Clearly we can swap $f(k)$ and $f(-k)$ without any trouble. Thus WLOG $f(k) > f(-k)$ for any $k=1,2,...,2562$. The expression evaluates to $$f(1)+f(2)+...+f(2562)-f(-1)-f(-2)-...-f(-2562).$$ Evidently it's minimized when $\{f(1),f(2),...,f(2562\}=\{1,2,...,2562\}$ and $\{f(-1),f(-2),...,f(-2562)\}=\{-1,-2,...,-2562\}$ which is basically the problem's condition. Solution 1 (Spiral Similarity): Let the incircle touches $AC,AB$ at $E,F$. Then just notice the spiral similarity $\triangle KBF\stackrel{+}{\sim}\triangle KCE$ thus $$\frac{KB}{KC}=\frac{BF}{CE}=\frac{BD}{DC}$$ or $KD$ bisects $\angle BKC$. This immediately implies $K,D,M$ are colinear. Solution 2 (Inversion): Again, let $E,F$ be the other two intouch points. Perform inversion around the incircle. We deduce the following facts. -$\triangle A'B'C'$ is medial triangle of $\triangle D'E'F'$. -$I$ is orthocenter of $\triangle A'B'C'$. -$M'$ is reflection of $I$ across $B'C'$. -$K'$ is foot from $D'$ to $E'F'$. This means points $\{K',D'\}$ and $\{I,M'\}$ are symmetric across $B'C'$. So $K'D'M'I$ is isosceles trapezoid which obviously cyclic. Inverting back, we find that $K,D,M$ are colinear. The answer is $\boxed{4038}$. To see the bound, note that each angle must be multiple of $\tfrac{\pi}{2019}$. Thus each angle has magnitude at most $\tfrac{2018\pi}{2019}$. Thus if the $n$-gon works, then $$\pi(n-2)\leqslant \tfrac{2018\pi}{2019}\cdot n\implies n\leqslant 4038.$$ For the construction, take a regular $4038$-gon and draw a line connecting the center to each of the $4038$ vertices. For each odd $n$, either $n$ or $n!-n$ works. To see why, let $n!+1=p$ be a prime. Then by a variant of Wilson's Theorem, $$n!(p-1-n)!\equiv (-1)^{n-1}\pmod p\implies (n!-n)! = (p-1-n)!\equiv -1\pmod p$$ thus $p\mid (n!-n)!+1$ so $n!-n$ works. 22 พฤษภาคม 2019 16:44 : ข้อความนี้ถูกแก้ไขแล้ว 3 ครั้ง, ครั้งล่าสุดโดยคุณ Pitchayut |
#9
|
||||
|
||||
คือเอาจริง ๆ ผมชอบอสมการนะครับ โดยเฉพาะปีนี้ค่อนข้างช็อคที่อสมการอยู่ข้อ 5 และก็ชอบมาก ๆ ด้วยเช่นกัน และเนื่องจาก @Pitchayut ได้ลง Solution ดี ๆ ไปแล้ว ดังนั้นผมจะมาเสนออีกแนวทางหนึ่ง นั่นคือ "การกระจาย"
อันนี้คือแค่มาบอกว่า บางทีตอนในห้องสอบถ้าคิดอะไรไม่ออก การกระจายอาจเป็นตัวเลือกหนึ่ง และอสมการของ TMO เป็นอสมการที่กระจายแล้วก็น่าจะออก อย่างข้อนี้ เท่าที่คุยกับผู้เข้าแข่งขันแต่ละคน คิดตั้งนานคิดไม่ออก ซึ่งก็มีทั้งที่ยอมกระจาย กับไม่ยอมกระจาย แต่ส่วนใหญ่ที่ลองกระจายก็ยอมแพ้กับการกระจายไป (กระจายไม่สุด) ซึ่งผมก็เข้าใจนะ เพราะผมเองก็ยังออกกลัว ๆ ที่จะกระจาย เพราะรูปที่เป็น cyclic แบบนี้มันดูเละมาก ๆ แต่ถ้าใจมันจะกระจาย อะไรก็ห้ามไม่อยู่แล้วล่ะ 555555 จากอสมการจะได้ว่าต้องพิสูจน์ว่า $$\sum_{cyc} [4a(2c+1)^2(2a+1)^2] \ge (2a+1)^2(2b+1)^2(2c+1)^2+\sum_{cyc}[(2c+1)^2(2a+1)^2]$$ ทีนี้กระจายไงดีไม่ให้น่าสับสน เราควรเขียนอะไรต่าง ๆ ให้อยู่ในรูป $\sum$ มาดูทีละก้อนก็จะได้ว่า \begin{align*}\sum_{cyc}[4a(2c+1)^2(2a+1)^2] &= \sum_{cyc}[4a(4ac+2a+2c+1)^2]\\&= \sum_{cyc} [4a(16a^2c^2+16a^2c+16ac^2+4a^2+4c^2+16ac+4a+4c+1)]\\&= \sum_{cyc} (64a^3c^2+64a^3c+64a^2c^2+16a^3+16ac^2+64a^2c+16a^2+16ac+4a) \\&= \sum_{cyc}(64a^2b^3+64ab^3+64a^2b^2+16a^3+16a^2b+64ab^2+16a^2+16ab+4a)\end{align*} ต่อไปก็ก้อนด้านขวาก็จะได้ว่า \begin{align*}\sum_{cyc}[(2c+1)^2(2a+1)^2]&=\sum_{cyc} (16a^2c^2+16a^2c+16ac^2+4a^2+4c^2+16ac+4a+4c+1)\\&=\sum_{cyc}(16a^2c^2+16a^2c+16ac^2+8a^2+16ac+8a+1)\\&=\sum_{cyc}(16a^2b^2+16ab ^2+16a^2b+8a^2+16ab+8a+1)\end{align*} และ \begin{align*}(2a+1)^2(2b+1)^2(2c+1)^2 &= \left(8abc+4\sum_{cyc}ab+2\sum_{cyc}a+1\right)^2 \\&= \left(4\sum_{cyc}ab+2\sum_{cyc}a+9\right)^2 \\&= 16\left(\sum_{cyc}ab\right)^2+16\left(\sum_{cyc}ab\right)\cdot \left(\sum_{cyc}a\right)+4\left(\sum_{cyc}a\right)^2+72\sum_{cyc}ab+36\sum_{cyc}a+81\\&=\left(16\sum_{cyc}a^2b^2+32abc\sum_{cyc} a\right)+\left(16\sum_{cyc}a^2b+16\sum_{cyc}ab^2+48abc\right)+\left(4\sum_{cyc}a^2+8\sum_{cyc}ab\right)+72\sum_{cyc}ab+36\sum_{c yc}a+81\\&=16\sum_{cyc}a^2b^2+16\sum_{cyc}a^2b+16\sum_{cyc}ab^2+4\sum_{cyc}a^2+80\sum_{cyc}ab+68\sum_{cyc}a+129\end{align*} เมื่อเอาก้อนที่ได้จากการกระจายไปแทนก็จะได้ว่าต้องพิสูจน์ว่า $$64\sum_{cyc}a^2b^3+64\sum_{cyc}ab^3+32\sum_{cyc}a^2b^2+16\sum_{cyc}a^3+32\sum_{cyc}ab^2+4\sum_{cyc}a^2\ge 16\sum_{cyc}a^2b+80\sum_{cyc}ab+72\sum_{cyc}a+132$$ หรือก็คือ $$16\sum_{cyc}a^2b^3+16\sum_{cyc}ab^3+8\sum_{cyc}a^2b^2+4\sum_{cyc}a^3+8\sum_{cyc}ab^2+\sum_{cyc}a^2\ge 4\sum_{cyc}a^2b+20\sum_{cyc}ab+18\sum_{cyc}a+33$$ ซึ่งพอถึงตรงนี้ก็พบว่ามันน่าจะพิสูจน์ได้ไม่ยากเพราะฝั่งซ้ายมีดีกรีที่สูง ในขณะที่ฝั่งขวามีดีกรีที่ต่ำ โดยพิสูจน์ได้คือ โดยอสมการ AM-GM จะได้ว่า $$4\sum_{cyc}a^3=4\sum_{cyc}\frac{a^3+a^3+b^3}{3}\ge 4 \sum_{cyc}a^2b$$ และ $$2\sum_{cyc}a^2b^2 = 2\sum_{cyc}\frac{a^2b^2+b^2c^2}{2} \ge 2\sum_{cyc} ab^2c = 2\sum_{cyc}a$$ และ $$4\sum_{cyc}a^2b^2 \ge \frac{4}{3}\left(\sum_{cyc}ab\right)^2\ge 4\sum_{cyc}{ab}$$ และ $$2\sum_{cyc}a^2b^2+8\sum_{cyc}ab^2+\sum_{cyc}a^2 \ge 6(abc)^\frac{4}{3}+24abc+3(abc)^\frac{2}{3}=33$$ โดยอสมการโคชี-ชวาร์ช ได้ว่า $$16\sum_{cyc}a^2b^3=16\sum_{cyc}\frac{a^2b^2}{ac}\ge 16\sum_{cyc}ab$$ และ $$16\sum_{cyc}ab^3=16\sum_{cyc}\frac{b^2}{c}\ge 16\sum_{cyc}a$$ เมื่อรวมอสมการทั้งหมดก็จะได้สิ่งที่ต้องการพิสูจน์พอดี ปล. กระจายแบบนี้ผมทำไปประมาณ 15-30 นาที ถ้าอยู่ในห้องสอบก็ต้องตัดสินใจดี ๆ นะครับก่อนกระจาย 555555555 21 พฤษภาคม 2019 21:49 : ข้อความนี้ถูกแก้ไขแล้ว 7 ครั้ง, ครั้งล่าสุดโดยคุณ NaPrai |
#10
|
||||
|
||||
สวัสดีครับ ไม่ได้แวะเวียนมาที่นี่ซะนาน อยู่มาตั้งแต่สมัยหนุ่ม ๆ 55555
เอา Solution ข้อ 8 แบบเต็ม ๆ และมีรูปมาแปะให้ครับ จริง ๆ ข้อนี้มีอีกหลาย Solution เลยนะครับ ดีจริง ๆ เลย นาน ๆ จะเจอโจทย์เรขา TMO ที่มันส์ขนาดนี้ ไม่แน่ปีนี้ Best Solution ของปีนี้อาจจะมาจากโจทย์เรขาก็ได้นะฮะ ป.ล. อยากเห็น Hell เอ้ย Heaven Edition ของข้อนี้จัง
__________________
...สีชมพูจะไม่จางด้วยเหงื่อ แต่จะจางด้วยนํ้าลาย... |
#11
|
|||
|
|||
มาสรุปผลของปีนี้นะครับ ข้อมูลทั้งหมดนี้เกิดจากการสอบถามคนที่อยู่ในงานมา ถ้าข้อมูลส่วนไหนผิดพลาดก็ขออภัยด้วยนะครับ
ทอง 7 เหรียญ ตัดที่ 40 คะแนน เงิน 16 เหรียญ ตัดที่ 28 คะแนน ทองแดง 25 เหรียญ ตัดที่ 15 คะแนน ปล. cutoff เงินกับทองแดงผมไม่ค่อยชัวร์นะครับ รบกวนคนที่ไปมาช่วยตรวจสอบด้วย ที่ 1 มีสองคน ได้ 44 คะแนน (จึงไม่มีที่ 2) ที่ 3 มีสองคน ได้ 42 คะแนน ไม่มีรางวัล Best Solution Best Problem ข้อ 4 ศูนย์โรงเรียนราชสีมาวิทยาลัย ที่มาของโจทย์ ข้อ 2 มาจากศูนย์มหาวิทยาลัยเชียงใหม่ ข้อ 4 มาจากศูนย์โรงเรียนราชสีมาวิทยาลัย ข้อ 8,9 มาจากศูนย์โรงเรียนสวนกุหลาบวิทยาลัย ที่เหลืออีก 6 ข้อ มาจากศูนย์โรงเรียนมหิดลวิทยานุสรณ์ทั้งหมด 22 พฤษภาคม 2019 20:37 : ข้อความนี้ถูกแก้ไขแล้ว 2 ครั้ง, ครั้งล่าสุดโดยคุณ Pitchayut |
#12
|
||||
|
||||
ปีนี้ศูนย์สวนกุหลาบ เปลี่ยนเป็นศูนย์อื่นแล้วเหรอครับ ไปดูในเพจ TMO มา คนที่ได้อันดับ 1,3 อยู่ศูนย์ที่ผมไม่รู้จักเลย
ปล. ปีนี้คะแนนน้อยกันจังเลยครับ ที่1ได้ 6 ข้อเอง
__________________
เหนือฟ้ายังมีอวกาศ 22 พฤษภาคม 2019 22:33 : ข้อความนี้ถูกแก้ไขแล้ว 1 ครั้ง, ครั้งล่าสุดโดยคุณ กขฃคฅฆง |
#13
|
||||
|
||||
ให้ $P(x,y)$ แทน $xf(y)+yf(x)\leqslant xy$ ทุก $x,y\in\mathbb{R}$ $P(1,0),P(-1,0)$ ทำให้ได้ว่า $f(0)\leqslant 0$ และ $-f(0)\leqslant 0$ ดังนั้น $f(0)=0$ $P(x,x)$ ได้ว่า $2xf(x)\leqslant x^2$ ทุก $x\in\mathbb{R}$ ดังนั้น $f(x)\leqslant \frac{x}{2} $ ทุก $x>0$ และ $f(x)\geqslant \frac{x}{2} $ ทุก $x<0$ เมื่อ $x<0$ และ $y>0$ ได้ว่า $xy\geqslant xf(y)+yf(x)\geqslant xf(y)+y\cdot\frac{x}{2}$ ทุก $x<0$ และ $y>0$ ดังนั้น $f(y)\geqslant\frac{y}{2}$ ทุก $y>0$ จาก $f(y)\leqslant \frac{y}{2}$ ทุก $y>0$ $\therefore f(x)=\frac{x}{2}$ ทุก $x>0$ ได้ว่า $xy\geqslant xf(y)+yf(x)\geqslant x\cdot\frac{y}{2}+yf(x)$ ทุก $x<0$ และ $y>0$ ดังนั้น $f(x)\leqslant\frac{x}{2}$ ทุก $x<0$ จาก $f(x)\geqslant \frac{x}{2}$ ทุก $x<0$ $\therefore f(x)=\frac{x}{2}$ ทุก $x<0$ จึงสรุปได้ว่า $f(x)=\frac{x}{2}$ ทุก $x\in\mathbb{R}$
__________________
Im korncrazy mathmania 23 พฤษภาคม 2019 22:51 : ข้อความนี้ถูกแก้ไขแล้ว 1 ครั้ง, ครั้งล่าสุดโดยคุณ จอมยุทธน้อย |
#14
|
||||
|
||||
อ้างอิง:
ให้ $\overrightarrow{MD}$ ตัด $\omega$ ที่ $P$ จะได้ว่า $MI^2=MB^2=MD\cdot MP$ ดังนั้น $M\hat ID=M\hat PI$ ทำให้ $I\hat PA=90^\circ$ นั่นคือ $P=K$ เพราะฉะนั้น $K,D,M$ อยู่บนเส้นตรงเดียวกัน ให้ $AM$ ตัด $BC$ ที่ $E$ inversion รอบวงกลมล้อมรอบสามเหลี่ยม $IBC$ จะได้ว่า $A=E$ และ $I=I$ ทำให้ $K=D$ ดังนั้น $K,D,M$ อยู่บนเส้นตรงเดียวกัน 24 พฤษภาคม 2019 01:47 : ข้อความนี้ถูกแก้ไขแล้ว 1 ครั้ง, ครั้งล่าสุดโดยคุณ Amankris |
หัวข้อคล้ายคลึงกัน | ||||
หัวข้อ | ผู้ตั้งหัวข้อ | ห้อง | คำตอบ | ข้อความล่าสุด |
TMO 15 Discussion | Pitchayut | ข้อสอบโอลิมปิก | 16 | 12 พฤษภาคม 2018 01:47 |
TMO 14 Discussion | Pitchayut | ข้อสอบโอลิมปิก | 15 | 18 พฤษภาคม 2017 17:21 |
TMO 13 Discussion | Beatmania | ข้อสอบโอลิมปิก | 32 | 29 กันยายน 2016 12:30 |
Topic for discussion | คนอ่อนคณิต | ทฤษฎีจำนวน | 1 | 06 มกราคม 2009 17:54 |
เครื่องมือของหัวข้อ | ค้นหาในหัวข้อนี้ |
|
|